Dinh ly muirhead va ung dung

5 8 0
Dinh ly muirhead va ung dung

Đang tải... (xem toàn văn)

Thông tin tài liệu

ĐỊNH LÝ MUIRHEAD VÀ ỨNG DỤNG ĐỊNH LÝ MUIRHEAD VÀ ỨNG DỤNG Trong bài viết này tôi xin giới thiệu về định lý Muirhead và ứng dụng của nó trong các bài toán chứng minh bất đẳng thức Trước hết ta phát biể[.]

ĐỊNH LÝ MUIRHEAD VÀ ỨNG DỤNG Trong viết xin giới thiệu định lý Muirhead ứng dụng tốn chứng minh bất đẳng thức Trước hết ta phát biểu định lý Muirhead dạng tổng quát cho n số: Cho n số biểu thức dạng Ta nói số dương Ký hiệu tổng đối xứng trội số nếu: ký hiệu: Khi đó: Đẳng thức xảy Cách chứng minh định lý Muirhead cho n số tương đối phức tạp Và thực tế ta không cần sử dụng đến trường hợp tổng quát Do khn khổ báo tơi xin sâu vào trường hợp thông dụng định lý Tôi chứng minh đưa ứng dụng định lý trường hợp Định lý Muirhead cho số: Cho số thực thoả mãn: Cho x, y, z số thực dương Khi đó: Đẳng thức xảy Chứng minh: Bổ đề: Cho số dương Khi CM: Không tổng quát ta giả sử với thoả mãn số x, y dương Dễ dàng có: ta có: Áp dụng: Ta xét trường hợp: a) Và Áp dụng bổ đề hai lần ta được: ( kí hiệu tổng hoán vị) b) Áp dụng bổ đề hai lần ta được: Định lý chứng minh Bây đưa ví dụ minh hoạ để bạn thấy ứng dụng định lý tốn cụ thể Ví dụ 1: CMR: ta có: CM: Bất đẳng thức cần chứng minh tương đương với: Do nên Bất đẳng thức cuối theo Muirhead Đẳng thức xảy Nhận xét: Bài tốn ta chọn đặt ta thu đuợc tốn IMO 1995 Ví dụ 2: Cho CMR: CM: Bất đẳng thức cần chứng minh tương đương với: Theo BĐT Cauchy-Schwarz để chứng minh toán rõ ràng ta cần chứng minh khẳng định sau: Bất đẳng thức cuối theo Muirhead Đẳng thức xảy Ví dụ 3: (USAMO 1997) CMR: ta có: CM: Bất đẳng thức cần chứng minh tương đương với: Bất đẳng thức cuối theo Muirhead Đẳng thức xảy Ví dụ 4: Cho CMR: CM: Vì nên để chứng minh tốn ta chứng minh khẳng định sau: Bất đẳng thức cuối theo Muirhead Đẳng thức xảy Qua ví dụ thấy để sử dụng hiệu định lý Muirhead chứng minh phải tìm cách đưa toán bất đẳng thức dạng đối xứng, đồng bậc Ngồi bạn nhận thấy toán giải định lý Muirhead thường khơng khó đường lối mà địi hỏi kỹ biến đổi tốn học xác Bên cạnh có bạn nhận xét định lý Muirhead hiệu lớp toán mà dấu đẳng thức xẩy tất biến Điều thực có ngoại lệ Ta xét trường hợp biến x, y, z tất nhiên điều kiện kèm phải Khi đẳng thức định lý Muirhead xẩy hoặc số x, y, z có số nhau, số cịn lại Ví dụ sau minh hoạ cho trường hợp này: Ví dụ 5: Cho thoả mãn CMR: CM: Bất đẳng thức cần chứng minh tương đương với: Rút gọn BĐT ta BĐT tương đương sau đây: Bất đẳng thức cuối theo Muirhead Đẳng thức xảy : Thông qua viết hy vọng bạn rút nhận xét kinh nghiệm cho riêng Để kết thúc xin gửi tới bạn số tập tương tự sau để bạn rèn luyện thêm: Bài 1: Cho thoả mãn CMR: Bài 2: (Iran 1996) Cho CMR: Bài 3: Cho độ dài cạnh tam giác CMR:

Ngày đăng: 13/04/2023, 07:45

Tài liệu cùng người dùng

Tài liệu liên quan